Error in Griffiths QM: completeness of stationary states

In summary, Griffiths claims that the stationary state solutions for a particle in an infinite potential well, which are functions of sine, are both orthogonal and complete. However, the speaker disagrees with the claim of completeness, stating that not all functions can be written in terms of these stationary states, giving the example of f(x)=cos(x). They also bring up the issue of boundary conditions and the definition of completeness in this context.
  • #1
Aziza
190
1
According to Griffiths QM book, after he derived the stationary state solutions to the Schrodinger equation for a particle in an infinite potential well, which are just functions of sine, he claims that these stationary solutions are orthogonal and complete.

I agree that they are orthogonal (since sin(nx) and sin(mx) are orthogonal for n!=m), but I definitely disagree that they are complete (meaning that ANY function f(x), odd or even, can be written in terms of these stationary states). For example, you definatelly cannot write f(x)=cos(x) using only a basis of sine functions.

Is there something I am missing here?



Thanks!
 
Last edited:
Physics news on Phys.org
  • #2
Aziza said:
According to Griffiths QM book, after he derived the stationary state solutions to the Schrodinger equation for a particle in an infinite potential well, which are just functions of sine,

Therefore the well has its left side at x = 0, correct?

I definitely disagree that they are complete (meaning that ANY function f(x), odd or even, can be written in terms of these stationary states). For example, you definatelly cannot write f(x)=cos(x) using only a basis of sine functions.

If the left side of the well is at x = 0, can cos(kx) be a valid solution?
 
  • #3
jtbell said:
Therefore the well has its left side at x = 0, correct?



If the left side of the well is at x = 0, can cos(kx) be a valid solution?

nope! cos cannot be a solution. I realized that maybe I am being too picky, but Griffiths' exact words are "The functions ψ_n (x) [where ψ_n (x) are the solutions to the infinite potential well] have some interesting properties...4. They are complete, in the sense that any other function f(x) can be expressed as a linear combination of them"

He goes on to say that this is Dirichlet's theorem.

By "any other function", I took it to literally mean any other function, not any other function that could possibly represent the wavefunction of a particle in infinite potential well.
Having studied Fourier analysis in detail, this is definitely not how I have seen the word "complete" and Dirichlet's theorem defined...
 
  • #4
Well Griffiths is being a little loose and he admits it when he comes to his discussion of the completeness of stationary states.

He is solving the infinite square well in the region (0, a). If you look at a good reference on Fourier series (For an intro QM level, I would recommend Mary Boas' "Mathematical Methods in the Physical Sciences", chapter 7 in the 3rd edition), you'll see that for functions which are periodic on an interval (0,a), the functions sin(nπx/a) form a basis for functions which are odd on that interval, while the functions cos(nπx/a) form a basis for even functions on that interval, and putting them together you can see that the functions e(inπx/a) form a complete basis for arbitrary functions (remembering all these functions must have period a).

The reason he forgets about the cos(nπx/a) terms is because the boundary condition ψ(x=0)=0 ensures that the solutions to the Schrodinger equation must be odd. Don't worry that the wavefunction doesn't really "continue" outside the well--just imagine that the solution repeats itself with period a, while respecting the boundary condition. [This is allowed because the potential is infinite outside the well.] So the sine functions he mentions form a complete basis for solutions to the Schrodinger equation since they must be odd with period a.

As a general recipe, you can represent pretty much any function in terms of sinusoidal functions with arbitrary period
f(x) = ∫ c(k) e(ikx) dk = ∫a(k) cos(kx) + b(k) sin(kx) dk

But if we limit to functions with period a, then we don't need any of the contributions from k≠nπ/a
thus if f(x)=f(x+a) for all x
then f(x) = Ʃn an cos(nπx/a) + bnsin(nπx/a)

For odd functions, you get no cos contributions, while for even functions, you get no sin contributions.

Edit: The font that PF appears in on my computer makes the n's look very similar to the π's. Sorry for that.
 
Last edited:
  • #5
Aziza said:
"The functions ψ_n (x) [where ψ_n (x) are the solutions to the infinite potential well] have some interesting properties...4. They are complete, in the sense that any other function f(x) can be expressed as a linear combination of them"
For a well-defined statement one has to specify the space (including boundary conditions) on which the Hamiltonian is self-adjoint and for which completeness shall hold
 
  • #6
Aziza said:
According to Griffiths QM book, after he derived the stationary state solutions to the Schrodinger equation for a particle in an infinite potential well, which are just functions of sine, he claims that these stationary solutions are orthogonal and complete.

I agree that they are orthogonal (since sin(nx) and sin(mx) are orthogonal for n!=m), but I definitely disagree that they are complete (meaning that ANY function f(x), odd or even, can be written in terms of these stationary states). For example, you definatelly cannot write f(x)=cos(x) using only a basis of sine functions.

Is there something I am missing here?
Inside the potential well, you can make a Fourier series out of sin(nx) functions, equal to cos(x) EVERYWHERE in the well except at the boundary.
 

Related to Error in Griffiths QM: completeness of stationary states

1. What is the completeness of stationary states in Griffiths QM?

The completeness of stationary states in Griffiths QM refers to the fact that any wave function can be written as a linear combination of stationary states. This means that a stationary state basis is complete and can be used to represent any state of a quantum system.

2. Why is the completeness of stationary states important in Griffiths QM?

The completeness of stationary states is important because it allows us to simplify the calculation of quantum mechanical systems. By using a complete set of stationary states, we can express any state of a system as a linear combination of these states, making the mathematical analysis more manageable.

3. How is the completeness of stationary states proven in Griffiths QM?

The completeness of stationary states is proven through the use of mathematical techniques, such as Fourier analysis and the spectral theorem. These techniques show that any wave function can be expressed as a linear combination of stationary states, thus proving their completeness.

4. Can the completeness of stationary states be extended to non-stationary states?

No, the completeness of stationary states only applies to stationary states. Non-stationary states, such as those that change over time, cannot be written as a linear combination of stationary states and therefore the completeness theorem does not apply to them.

5. How does the completeness of stationary states relate to the uncertainty principle in Griffiths QM?

The completeness of stationary states is closely related to the uncertainty principle in Griffiths QM. The uncertainty principle states that it is impossible to simultaneously know the exact position and momentum of a particle. This is due to the fact that stationary states, which have definite energy, do not have definite position or momentum. This relationship highlights the limitations of the completeness of stationary states and the uncertainty inherent in quantum mechanical systems.

Similar threads

Replies
10
Views
3K
Replies
5
Views
5K
  • Quantum Physics
Replies
2
Views
2K
Replies
3
Views
2K
Replies
2
Views
619
  • Quantum Physics
Replies
6
Views
905
  • Introductory Physics Homework Help
Replies
2
Views
559
  • Advanced Physics Homework Help
Replies
3
Views
1K
  • Quantum Physics
Replies
2
Views
1K
Replies
9
Views
1K
Back
Top